• Non ci sono risultati.

1. Let P = (1, 2, 1) and let R be the line {(1, 5, −1) + t(1, 1, −1)}. Compute the distance between P and L and find the point of L which is nearest to P .

N/A
N/A
Protected

Academic year: 2021

Condividi "1. Let P = (1, 2, 1) and let R be the line {(1, 5, −1) + t(1, 1, −1)}. Compute the distance between P and L and find the point of L which is nearest to P ."

Copied!
3
0
0

Testo completo

(1)

Linear Algebra and Geometry. Written test of september 12, 2013.

1. Let P = (1, 2, 1) and let R be the line {(1, 5, −1) + t(1, 1, −1)}. Compute the distance between P and L and find the point of L which is nearest to P .

Solution. Let u = (1, 2, 1) − (1, 5, −1) = (0, −3, 2). We have that the projection of u along L((1, 1, −1) is pr(u) = −5/3(1, 1, −1). Therefore the nearest point is

H = (1, 5, −1) − 5/3(1, 1, −1) The distance is

k u − pr(u) k=k (0, −3, 2) + 5/3(1, 1, −1) k=k 1/3(5, −4, −1) k= √ 42/3

2. A particle moves along the ellipse 2x 2 + y 2 = 1 with position vector r(t) = (x(t), y(t)).

The motion is such that x 0 (t) = −4y(t) for each t.

(a) Prove that y 0 (t) is proportional to x(t) and find the factor of proportionality;

(b) How much time is required for the particle to go once around the ellipse?

Solution. We have that 2x(t) 2 + y(t) 2 = 1 for each t. Differentiating we get that 4x(t)x 0 (t) + 2y(t)y 0 (t) = 0. Since x 0 (t) = −4y(t) we get −16x(t)y(t) + 2(y(t)y 0 (t) = 2y(t)(−8x(t) + y 0 (t)) ≡ 0. Since y(t) is non-zero except for some isolated points of t we conclude, from the continuity of y 0 (t), that y 0 (t) = 8x(t) for each t.

(b) We have that (x 0 (t), y 0 (t) = (−4y(t), 8x(t)) Differentiating another time we get

(x 00 (t), y 00 (t)) = (−32x(t), −32y(t)). It turns out that both x(t) and y(t) are solutions of the equation f 00 = −32f . Therefore they are of the form a sin 4 √

2t + b cos 4 √

2t. Hence the time to go once around the ellipse is T = 2π/(4 √

2).

3. In V 4 , with the usual dot product, let W = L((1, 1, −1, 0), (1, 0, 0, 1)) and let v = (1, 1, 1, 0).

(a) Find a vector w ∈ W and a vector u ∈ W such that v = w + u.

(b) Is it possible to find two other vectors w 0 ∈ W and u 0 ∈ W , different from w and u, such that v = w 0 + u 0 ?

Solution. (b) By the Theorem on Orthogonal Decompositions, vectors w and u as required do exist, and they are unique. Therefore the answer to (b) is NO.

(a) By the Theorem on Orthogonal Decompositions, we have that w must be the projection of v onto W , which will be denoted pr W (v) , and u = v − pr W (v). Therefore we have only to compute the projection of v onto W . In order to do that we have first to orthogonalize the given basis of W . We take w 1 = (1, 1, −1, 0). Moreover we have that

(1, 0, 0, 1) − pr L(v

1

) ((1, 0, 0, 1)) = (1, 0, 0, 1) − (1/3)(1, 1, −1, 0) = (1/3)(2, −1, 1, 3) Therefore, to simplify the calculation, we can take w 2 = (2, −1, 1, 3). Finally, se have that

pr W (v) = pr L(w

1

) (v) + pr L(w

2

) (v) = (1/3)(1, 1, −1, 0) + (6/5)(2, −1, 1, 3)

1

(2)

As already mentioned before u := v − pr W (v) belongs to W .

4. Let T : V 3 → V 3 be the linear transformation such that T ((1, 0, 1)) = (1, 1, 2), T ((1, 0, −1) = (0, 2, 3) and T ((1, 2, −1)) = (1, 3, 5).

(a) Find the dimension and a basis of the range T (V 3 );

(b) find the dimension and a basis of the null-space N (T );

(c) find the matrix of T with respect to the canonical basis of V 3 .

Solution. (a) We denote v 1 = (1, 0, 1), v 2 = (1, 0, −1) and v 3 = (1, 2, −1). Since they are independent (easily seen), they form a basis B of V 3 . An easy computation shows that T (v 1 ), T (v 2 ) and T (v 3 ) are not independent. Since the first two are independent, it follows that rk(T ) = 2 and that {(1, 1, 2), (0, 2, 3)} is a basis of T (V 3 ).

(b) By the nullity + rank Theorem, the nullity is 1. To find a basis, the simpler way is to note that T (v 1 ) + T (v 2 ) = T (v 3 ). This means that T (v 1 + v 2 − v 3 ) = 0, that is v 1 + v 2 − v 3 ∈ N (T ). Since we know that dim N (T ) = 1, we conclude that {v 1 + v 2 − v 3 } is a basis of N (T ).

If one does not see that T (v 1 ) + T (v 2 ) = T (v 3 ), one can argue as follows. We have that

B = m B E (T ) =

1 0 1 1 2 3 2 3 5

The solutions of the system B

 x 0 y 0 z 0

 =

 0 0 0

 are the components – with respect to the basis B – of vectors v such that T (v) = 0. It is easily seen that the solutions of the above system are the triples (x 0 , y 0 z 0 ) such that x 0 = −z 0 and y 0 = −z 0 , that is of the form λ(1, 1, −1) for λ ∈ R. We conclude, as above, that N (T ) is spanned by 1 v 1 + 1 v 2 + (−1) v 3 .

(c) We have that

m E E (T ) = m B E (T ) m E B (id)

(you can see this by writing down the corresponding diagram of linear transformations).

Since m E B (id) = m B E (id) −1 = C −1 , where

C =

1 1 1

0 0 2

1 −1 −1

we finally have

m E E (T ) = B C −1

5. (a) Find all real 3 × 3 symmetric matrices A such that: 2 is a double eigenvalue of A, det A = −12 and L((1, 1, 1)) is an eigenspace of A.

(b) For a matrix A as in (a), let us consider the function Q A : V 3 → R defined by

2

(3)

Q A (X) = X A X t . Find a triple ¯ X = (¯ x 1 , ¯ x 2 , ¯ x 3 ) such that Q A ( ¯ X) is positive and a triple Y = (¯ ¯ y 1 , ¯ y 2 , ¯ y 3 ) such that Q A ( ¯ Y ) is negative.

Solution. (a) The matrix A must have another (simple) eigenvalue. Since the product of (possibly coinciding) eigenvalues of A equals det A = −12, this simple eigenvalue is

−3. Since A is symmetric, it is diagonalizable, hence the eigenspace corresponding to the double eigenvalue must have dimension 2. Therefore L((1, 1, 1)) must be the eigenspace E(−3). Since A is symmetric, the eigenspace E(2) must be E(−3) , hence it is defined by the equation x + y + z = 0. Hence E(−3) = L((−1, 1, 0), (−1, 0, 1)). Putting together the basis of E(2) and the basis of E(−3) we get a basis B of V 3 . We consider the matrix C = m B E (id) =

−1 −1 1

1 0 1

0 1 1

. We know that diag(2, 2, −3) = C −1 A C. Therefore the final answer to A is the following: there is one and only one such matrix A, namely

A = C diag(2, 2, −3) C −1

(b) Note that the function Q A is the quadratic form associated to the matrix A. Certainly an eigenvector ¯ X of 2 – for example ¯ X = (−1, 1, 0) – is such that Q A ( ¯ X) > 0. Indeed Q A ( ¯ X) = ¯ X A ¯ X t = ¯ X 2 ¯ X t = 2 ¯ X · ¯ X > 0. By the same reason, an eigenvector ¯ Y of −3, for example (1, 1, 1), is such that Q( ¯ Y ) < 0.

3

Riferimenti

Documenti correlati

Find for which values of t and a there is a unique solution, no solution, infinitely many

Find all points of maximum and all points

We compute the cartesian equation of M.. See Test 3 of the

NOTE: In the solution of a given exercise you must (briefly) explain the line of your argument and show the main points of your calculations1. Solutions without adequate

Let V be the linear space of polynomials of degree at most 3.. Describe the points of minimum and the points

Indeed we know that the maximal eigenvalue is the max- imum value taken by the quadratic form on the unit sphere (that is, the set of all

At time t 0 find: the unit normal vector, the equation of the osculating plane, the radius of curvature, the center of curvature, a parametrization of the osculating

(b) If the answer is yes, show an example of such a vector